Mehrfachintegrale Aufgabenstellung

Aufrufe: 1073     Aktiv: 06.10.2021 um 13:34

0


Guten Tag, ich stehe hier vor einem Problem... Ich weiß wie man Mehrfachintegrale berechnet, nur verstehe ich nicht ganz welche Grenzen ich einsetzten soll. Bedeutet hier der Bereich B folgendes (also meine Überlegung): 

 Das dx berechne ich von Obergrenze: 2 bis Untergrenze -4.

Und das dy von Obergrenze: x^2+2 bis Untergrenze (x/2)-2. ( hier hätte ich die Ungleichung 2y >= x-4 einfach nach y aufeglöst)

Aber natürlich müsse ich dann vorher nach dy integrieren (ist das Erlaubt?), damit ja dann am ende keine Variablen stehen bleiben... Stimmt das so oder bin ich da am Holzewg?

EDIT vom 05.10.2021 um 09:33:

Nachtrag: Habe hier mal den Integrationsbereich den ich skizziert habe

EDIT vom 05.10.2021 um 18:40:

Nachtrag: ich habe hier jetzt die originale Aufgabenstellung gepostet, bei der ich am verzweifeln bin...

mfg Xaver

EDIT vom 06.10.2021 um 00:08:

Neues Bild vom IB

Diese Frage melden
gefragt

Student, Punkte: 80

 
Kommentar schreiben
3 Antworten
1
Genau, du musst dann zuerst über y integrieren, hört sich aber sonst soweit korrekt an.
\[\int_{-4}^2 \int_{\frac{x-4}{2}}^{x^2+2} 2x e^y\; \mathrm{d}y\mathrm{d}x\]
Wenn du zuerst über \(x\) integrieren wollen würdest, müsstest du die Definition des Gebietes auch dementsprechend umformen (1), also \(y_1 \le y \le y_2 \;\land\; g(y) \le x \le h(y)\).


(1) Hier wegen dem quadratischen Term nicht möglich, es sei denn man teilt B in 2 Gebiete (\(\left\{ x \in B \; :\; x<0 \right\}\) und \(\left\{ x \in B \; :\; x\ge 0 \right\}\)) und \(y\) in weitere Intervalle auf - es wird aufjedenfall deutlich komplexer.
Diese Antwort melden
geantwortet

Student, Punkte: 1.05K

 

Vielen Dank!   ─   xaverhauer 02.10.2021 um 15:35

Kommentar schreiben

1
Dein Vorgehen ist genau richtig, berechne also $\int\limits_{-4}^2\int\limits_{\frac{x}2-2}^{x^2+2} f(x,y)\, dy\, dx$, genauer also: $\int\limits_{-4}^2(\int\limits_{\frac{x}2-2}^{x^2+2} f(x,y)\, dy)\, dx$. Zuerst nach y, dann nach x.
In der anderen Reihenfolge würde es auch gar nicht gehen, denn dann wäre ja beim inneren Integral das x gleichzeitig Integrationsvariable und auch in den Grenzen auftauchen, das geht nicht.
Eine Skizze vom Integrationsgebiet ist auch oft hilfreich.
Diese Antwort melden
geantwortet

Lehrer/Professor, Punkte: 38.93K

 

Vielen Dank!
  ─   xaverhauer 02.10.2021 um 15:35

Jetzt habe ich ein anderes Problem bei einer gleichen Aufgabenstellung, nur dass ich für das dy die Grenzen habe, aber für das dx nicht, weil der Bereich wie folgt definiert ist: x^2+y^2 <= 4 und x <= 2+y .... Jetzt habe ich hier das Problem dass ich ja einmal ein <= und einmal ein >= brauche für die Grenzen... Die Definition von dem dy lautet: -2-x <= y <= 2-x
Aber solche Ungleichzeichen (<=) drehen sich ja nur um wenn man alles mit *(-1) multipliziert, was dann aber wiederum ein -x geben würde... Wie gehe ich hier nun am besten vor? Eigentlich wollte ich einfach die x^2+y^2 <= 4 umstellen auf x, aber dann ist eben immernoch <=
  ─   xaverhauer 02.10.2021 um 18:10

Nein das hab ich leider nicht gelernt wie man bei so etwas eine Skizze erstellt...   ─   xaverhauer 02.10.2021 um 23:45

1
Ich geb dir mal einen Tipp: \(x^2 + y^2\) ist das Quadrat der Euklidischen Norm (Länge) im \(\mathbb{R}^2\). Damit beschreibt \(x^2 + y^2 \le c\) eine Kreisfläche um den Ursprung mit Radius \(\sqrt{c}\).
Kannst du jetzt eine Skizze erstellen?
  ─   posix 03.10.2021 um 08:38

1
Diese Ungleichung für einen Kreis (oder auch im höherdimensionalen Kugel/...) wirst du sicherlich noch öfters sehen, es wäre sehr lohnenswert das wirklich zu verstehen.   ─   posix 03.10.2021 um 08:41

Danke schonmal für die Antworten, und Entschuldigung das ich mich jetzt erst melde, nur jetzt ist wieder Unistress... Ich habe mich da jetzt reingelesen, und habe auch auf Desmos mir die Integrationsbereiche skizziert.(weils einfach schnell geht und man experimentieren kann). x^2+y^2 <= 4 ist ein Kreis mit dem Radius 2, das ist mir mittlerweile "logisch" warum... Nur bei x <= 2+y bin ich mir nicht sicher, das ist einfach nur eine lineare Funktion die bei (0/-2) und (2/0) durchgeht, und somit einen Teil des Kreises durchschneidet... dann hab ich mir noch gedacht ich könnte die Ungleichung umstellen auf x-y <= 2 , und das ist genau das Gleiche... also ist nun mein Integrationsbereich die Überschneidung von dem Kreis und der linearen Funktion? Wenn ja, verstehe ich immer noch nicht woher ich dann die Grenzen erhalten soll.... das wäre ja der Flächeninhalt von einem Kreis minus einem Stück davon abgeschnitten.... mfg xaver   ─   xaverhauer 04.10.2021 um 20:29

Ok vielen Danke für die Antwort... ich habe mir das jetzt oft durchgelesen und recherchierte und verstanden was Sie meinen mit den senkrechten und waagrechten Streifen. Dann habe mich mir noch einmal den IB skizziert und zwar von allen 4 Bedingungen, also von x^2+y^2 <= 4 und x <= 2+y und -2-x <= y <= 2-x, und habe jetzt herausgefunden dass die Schnittmenge ein Rechteck im Kreis ist, was auf einer Seite "offen" ist und dort durch den Rand der Kurve begrenzt ist und auch nicht "gerade" im x,y Koordinatensystem liegt, sondern quasi 90° verschoben. Ich würde nun die Grenzen von dx mit 2-y und -2-y beschreiben.... weil ich das wenn ich richtig denke vertauschen muss? und dann würde ich als erstes nach dx, und dann bei dy würde ich die Grenzen von 2 bis -2, aber was passiert mit dem Kurvenstück, das passt ja dann auch wieder nicht... also ich bin hier ziemlich am Verzweifeln :=(   ─   xaverhauer 05.10.2021 um 08:11

Ok, habe Ihnen nun die originale Aufgabenstellung gepostet....   ─   xaverhauer 05.10.2021 um 18:41

Ok, vielen Dank.... Ich bin mir mit den Grenzen immernoch unsicher.... kann es sein dass dann das Doppelint. lautet:
12xy dy dx mit den Grenzen:

dy = Obergrenze: 2-x und Untergrenze -2-x
dx = Obergrenze: 2 und Untergrenze -2

aber was ich nicht verstehe ist, wie ich den Teil in die Grenzen bekommer der qasui der 1/4 Kreis ist...

Ich hätte noch ein dumme Idee die glaube ich kompliziert ist, indem ich den 1/4 Kreis mit r*dr*dphi berechne bzw integriere und die anderen "3 Dreiecke" indem ich mir da 1 Dreieck ausrechne und dann mal 3 rechne... aber das muss doch viel einfach gehen oder nicht?
Ich bin mir da mit den Grenzen einfach sehr unsicher....
  ─   xaverhauer 05.10.2021 um 22:43

Ok, danke. Ich hab jetzt das Bild nochmal neu gezeichnet, und auch die senkrechten Linien eingetragen. Nur irgendwie komme ich trotzdem noch nicht weiter, ich komm mir schon so dumm vor. Wenn ich jetzt das ganze in y Richtung integrieren will, dann habe ich (von links nach rechts mit dem Streifen) einmal die Grenze von -2-x und einmal oben das von der Kreiskurve (bei x im negativen) und dann nach x=0, also im positiven, einmal das 2+y und einmal das 2-x... und wurzel(4-x^2) wäre die Raumkurve, aber nur die "obere Hälfte"... ich verstehe jetzt nicht wie ich diese 4 Sachen zusammenknüpfen kann, damit ich auf die Grenzen von dy komme... tut mir echt schon leid dass ich so nerve aber ich wills echt verstehen, aber steh komplett am Holzweg irgendwie...   ─   xaverhauer 06.10.2021 um 00:06

Ok, danke..
Dann komme ich auf folgenden Grenzen:
Im Bereich von x=-2 bis x=0 für dy: Untergrenze -2-x und Obergrenze Wurzel(4-x^2)

Im Bereich x=0 bis x=2 für dy: Untergrenze -2+x und Obergrenze 2-x

Stimmt das jetzt? Muss ich also mit zwei Doppelintegralen hier arbeiten und die Endergebnisse addieren?

Dx würde dann einmal von Untergrenze-2 Obergrenze 0

Und einmal von Untergrenze 0 Obergrenze 2
  ─   xaverhauer 06.10.2021 um 13:00

Ok ja dann habe ich jetzt eeeeendlich alles verstanden 😍 danke...

Ja also ich hatte um ehrlich zu sein viele Probleme, erstens mal den IB zu skizzieren, dann zb auch noch dass ich hier aus einem Integral zwei machen kann usw usw... also habe viel dazugelernt
  ─   xaverhauer 06.10.2021 um 13:25

Leider scheint diese Antwort Unstimmigkeiten zu enthalten und muss korrigiert werden. Mikn wurde bereits informiert.